U2-Robbins- C13: WBC, Lymph Node, Spleen, Thymus Flashcards

1
Q

A 15-year-old boy visits his physician because of high fever of 10 days’ duration. Physical examination shows a
temperature of 38°C. He has scattered petechial hemorrhages on the trunk and extremities. There is no enlargement of
liver, spleen, or lymph nodes. The CBC shows hemoglobin, 13.2 g/dL; hematocrit, 38.9%; MCV, 93 μm3; platelet count,
175,000/mm3; and WBC count, 1850/mm3 with 1% segmented neutrophils, 98% lymphocytes, and 1% monocytes. Bone
marrow biopsy examination does not show any abnormal cells. Which of the following is the most likely diagnosis?
□ (A) Acute lymphoblastic leukemia
□ (B) Acute myelogenous leukemia
□ (C) Aplastic anemia
□ (D) Idiopathic thrombocytopenic purpura
□ (E) Overwhelming bacterial infection

A

(E) The major finding in this patient is marked granulocytopenia. All that remains on the peripheral smear are

mononuclear cells (remember to multiply the percentages in the differential by the total WBC count to get the absolute

values; rather than one cell line being overrepresented, another may be nearly missing). Accelerated removal or

destruction of neutrophils could account for the selective absence of granulocytes in this case. Overwhelming infections

cause increased peripheral use of neutrophils at sites of inflammation. Petechial hemorrhages also can occur in

overwhelming bacterial infections, such as those caused by Neisseria meningitidis. Bleeding is unlikely to be caused by

thrombocytopenia because the platelet count is normal. Normal bone marrow findings exclude acute lymphoid or myeloid

leukemia. In aplastic anemia, the marrow is poorly cellular, and there is a reduction in RBCs, WBCs, and platelet

production

How well did you know this?
1
Not at all
2
3
4
5
Perfectly
2
Q

2 A 9-year-old boy is taken to his pediatrician because of a generalized seborrheic skin eruption and fever. He has been
diagnosed and treated for otitis media several times in the past year. On physical examination, he has mild
lymphadenopathy, hepatomegaly, and splenomegaly. The electron micrograph shown in the figure was taken from a mass
lesion involving the mastoid bone. What is the most likely diagnosis?
□ (A) Acute lymphoblastic leukemia
□ (B) Multiple myeloma
□ (C) Hodgkin lymphoma, mixed cellularity type
□ (D) Langerhans cell histiocytosis
□ (E) Disseminated tuberculosis

A

(D) Shown here are the famous rodlike tubular Birbeck granules, with the characteristic periodicity seen in Langerhans

cell proliferations. In this case, the skin eruptions, organomegaly, and lesion in the mastoid suggest infiltrates in multiple

organs. The diagnosis is multifocal Langerhans cell histiocytosis, a disease most often seen in children. In half of these

cases, exophthalmos occurs, and involvement of the hypothalamus and pituitary stalk leads to diabetes insipidus; these

findings are called Hand-Schüller-Christian disease. Acute lymphoblastic leukemia in children can involve the marrow, but

does not produce skin or bone lesions. Myeloma is a disease of adults that can produce lytic bone lesions, but not skin

lesions. Hodgkin lymphoma is seen in young adults and does not produce skin lesions or bone lesions. Tuberculosis can

produce granulomatous disease with bony destruction, but the macrophages present in the granulomas are epithelioid

macrophages that do not have Birbeck granules.

How well did you know this?
1
Not at all
2
3
4
5
Perfectly
3
Q

A 67-year-old man has had increasing weakness, fatigue, and weight loss over the past 5 months. He now has
decreasing vision in both eyes and has headaches and dizziness. His hands are sensitive to cold. On physical
examination, he has generalized lymphadenopathy and hepatosplenomegaly. Laboratory studies indicate
hyperproteinemia with a serum protein level of 15.5 g/dL and albumin concentration of 3.2 g/dL. A bone marrow biopsy is
performed, and microscopic examination of the specimen shows infiltration of small plasmacytoid lymphoid cells with
Russell bodies in the cytoplasm. Which of the following findings is most likely to be reported for this patient?
□ (A) Monoclonal IgM spike in serum
□ (B) WBC count of 255,000/mm3
Robbins & Cotran Review of Pathology Pg. 242
□ (C) Hypercalcemia
□ (D) Bence Jones proteinuria
□ (E) Karyotype with t(14;18) translocation

A

(A) This patient has symptoms of hyperviscosity syndrome, including visual disturbances, dizziness, and headaches. He

also seems to have Raynaud phenomena. His bone marrow is infiltrated with plasmacytoid lymphocytes that have stored

immunoglobulins in the cytoplasm (Russell bodies). All of these findings suggest that the patient has lymphoplasmacytic

lymphoma (Waldenström macroglobulinemia). In this disorder, neoplastic B cells differentiate to IgM-producing cells; there

is a monoclonal IgM spike in the serum. These IgM molecules aggregate and produce hyperviscosity, and some of them

agglutinate at low temperatures and produce cold agglutinin disease. There is typically no leukemic phase to Waldenström

macroglobulinemia. Myeloma, which is typically accompanied by a monoclonal gammopathy, most often does not cause

liver and spleen enlargement, and morphologically, the cells resemble plasma cells. Hypercalcemia occurs with myeloma

because of bone destruction, and punched-out lytic lesions are typical of multiple myeloma. Light chains in urine (Bence

Jones proteins) also are a feature of multiple myeloma. A t(14;18) translocation is characteristic of a follicular lymphoma.

How well did you know this?
1
Not at all
2
3
4
5
Perfectly
4
Q

A 37-year-old woman visits her physician because of a cough and fever of 1 week’s duration. On physical examination,
her temperature is 38.3°C. She has diffuse crackles in all lung fields. A chest radiograph shows bilateral extensive
infiltrates. CBC shows hemoglobin, 13.9 g/dL; hematocrit, 42%; MCV, 89 μm3; platelet count, 210,000/mm3; and WBC
count, 56,000/mm3 with 63% segmented neutrophils, 15% bands, 6% metamyelocytes, 3% myelocytes, 1% blasts, 8%
lymphocytes, 2% monocytes, and 2% eosinophils. The peripheral blood leukocyte alkaline phosphatase score is
increased. Which of the following is the most likely diagnosis?
□ (A) Chronic myelogenous leukemia
□ (B) Hairy cell leukemia
□ (C) Hodgkin lymphoma, lymphocyte depletion type
□ (D) Leukemoid reaction
□ (E) Acute lymphoblastic leukemia

A

(D) Marked leukocytosis and immature myeloid cells in the peripheral blood can represent an exaggerated response to

infection (leukemoid reaction), or they can be a manifestation of chronic myelogenous leukemia (CML).The leukocyte

alkaline phosphatase score is high in the more differentiated cell population seen in reactive leukocytosis, whereas in

CML, the leukocyte alkaline phosphatase score is low. The Philadelphia chromosome (universally present in CML) is

lacking in patients with leukemoid reactions. Hairy cell leukemia is accompanied by peripheral blood leukocytes that mark

with tartrate-resistant acid phosphatase. Hodgkin lymphoma is not characterized by an increased WBC count. Acute

lymphoblastic leukemia is a disease of children and young adults, and the lymphoid cells do not have leukocyte alkaline

phosphatase

How well did you know this?
1
Not at all
2
3
4
5
Perfectly
5
Q

A 12-year-old boy is taken to the physician because he has had increasing abdominal distention and pain for the past 3
days. Physical examination shows lower abdominal tenderness, and the abdomen is tympanitic with reduced bowel
sounds. An abdominal CT scan shows a 7-cm mass involving the region of the ileocecal valve. Surgery is performed to
remove the mass. Histologic examination of the mass shows sheets of intermediate-sized lymphoid cells, with nuclei
having coarse chromatin, several nucleoli, and many mitoses. A bone marrow biopsy sample is negative for this cell
population. Cytogenetic analysis of the cells from the mass shows a t(8;14) karyotype. Flow cytometric analysis reveals
40% of the cells are in S phase. The tumor shrinks dramatically in size after a course of chemotherapy. Which of the
following is the most likely diagnosis?
□ (A) Diffuse large B-cell lymphoma
□ (B) Follicular lymphoma
□ (C) Acute lymphoblastic leukemia
□ (D) Plasmacytoma
□ (E) Burkitt lymphoma

A

(E) Burkitt and Burkitt-like lymphomas can be seen sporadically (in young individuals), in an endemic form in Africa (in

children), and in association with HIV infection. All forms are highly associated with translocations of the MYC gene on

chromosome 8. In the African form and in HIV-infected patients, the cells are latently infected with Epstein-Barr virus

(EBV), but sporadic cases are negative for EBV. This form of lymphoma is typically extranodal. Diffuse large cell

lymphomas are most common in adults, as are follicular lymphomas; they do not carry the t(8;14) translocation. Acute

lymphoblastic lymphomas can be seen in boys this age, but the mass is in the mediastinum, and the lymphoid cells are T

cells. Plasmacytomas appear in older adults and are unlikely to produce an abdominal mass. Because of the high growth

fraction (40% in this case), Burkitt lymphomas respond very well to chemotherapy that includes cycle acting agents. By

contrast, slow-growing tumors with a low growth fraction are more indolent and less responsive to chemotherapy.

How well did you know this?
1
Not at all
2
3
4
5
Perfectly
6
Q

A 53-year-old man comes to his physician because he felt a lump near his shoulder 1 week ago. On physical
examination, there is an enlarged, nontender, supraclavicular lymph node and enlargement of the Waldeyer ring of
oropharyngeal lymphoid tissue. There is no hepatosplenomegaly. CBC is normal except for findings of mild anemia. A
lymph node biopsy specimen shows replacement by a monomorphous population of large lymphoid cells with enlarged
nuclei and prominent nucleoli. Immunohistochemical staining and flow cytometry of the node indicates that most lymphoid
cells are CD19+, CD10+, CD3−, CD15−, and terminal deoxynucleotidyl transferase negative (TdT−). Which of the
following is the most likely diagnosis?
□ (A) Chronic lymphadenitis
□ (B) Diffuse large B-cell lymphoma
□ (C) Hodgkin lymphoma
□ (D) Lymphoblastic lymphoma
□ (E) Small lymphocytic lymphoma

A

(B) Diffuse large B-cell lymphoma occurs in older individuals and frequently manifests as localized disease with

extranodal involvement, particularly of the Waldeyer ring. The staining pattern indicates a B-cell proliferation (CD19+,

CD10+). T-cell (CD3) and monocytic (CD15) markers are absent. TdT can be expressed in B lineage cells at an earlier

stage of maturation. Small lymphocytic lymphoma also is a B-cell neoplasm, but it manifests with widespread

lymphadenopathy, liver and spleen enlargement, and lymphocytosis. Lymphoblastic lymphoma is a T-cell neoplasm that

occurs typically in the mediastinum of children. Hodgkin lymphoma is characterized by Reed-Sternberg cells. In chronic

lymphadenitis, the lymph node has many cell types—macrophages, lymphocytes, and plasma cells. A monomorphous

infiltrate is typical of non-Hodgkin lymphomas

How well did you know this?
1
Not at all
2
3
4
5
Perfectly
7
Q

A 50-year-old man has had headache, dizziness, and fatigue for the past 3 months. His friends have been commenting
about his increasingly ruddy complexion. He also has experienced generalized and severe pruritus, particularly when
showering. He notes that his stools are dark. On physical examination, he is afebrile, and his blood pressure is
165/90 mm Hg. There is no hepatosplenomegaly or lymphadenopathy. A stool sample is positive for occult blood. CBC
shows hemoglobin, 22.3 g/dL; hematocrit, 67.1%; MCV, 94 μm3; platelet count, 453,000/mm3; and WBC count, 7800/mm3.
The serum erythropoietin level is very low. What is the most likely diagnosis?
□ (A) Myelodysplastic syndrome
□ (B) Essential thrombocytosis
□ (C) Chronic myelogenous leukemia
□ (D) Erythroleukemia
□ (E) Polycythemia vera

A

(E) This patient has polycythemia vera, a myeloproliferative disorder characterized by an increased RBC mass, with

hematocrit concentrations typically exceeding 60%. Although the increased RBC mass is responsible for most of the

symptoms and signs, these patients also have thrombocytosis and granulocytosis. This occurs because, similar to other

myeloproliferative disorders, polycythemia vera results from transformation of a multipotent stem cell. The high hematocrit

concentration causes an increase in blood volume and distention of blood vessels. The neoplastic erythroid progenitor

cells require extremely small amounts of erythropoietin for survival and proliferation; the levels of erythropoietin are

virtually undetectable in polycythemia vera. When combined with abnormal platelet function, this condition predisposes the

patient to bleeding. Abnormal platelet function also can predispose to thrombosis. The pruritus and peptic ulceration most

Robbins & Cotran Review of Pathology Pg. 261

likely are the result of the histamine release from basophils. In some patients, the disease “burns out” to myelofibrosis. A

few patients “blast out” into acute myelogenous leukemia, and other patients develop chronic myelogenous leukemia.

Myelodysplastic syndromes and myeloproliferative disorders, such as essential thrombocytosis, are not accompanied by

such an increase in RBC mass. Erythroleukemia typically is not accompanied by such a high hematocrit concentration

because leukemic erythroid progenitors do not differentiate into mature RBCs.

How well did you know this?
1
Not at all
2
3
4
5
Perfectly
8
Q

A 50-year-old man was diagnosed with a diffuse large B-cell lymphoma. He underwent intensive chemotherapy, and a
complete remission was achieved for 7 years. He now reports fatigue and recurrent pulmonary and urinary tract infections
over the past 4 months. Physical examination shows no masses, lymphadenopathy, or hepatosplenomegaly. CBC shows
hemoglobin, 8.7 g/dL; hematocrit, 25.2%; MCV, 88 μm3; platelet count, 67,000/mm3; and WBC count, 2300/mm3 with 15%
segmented neutrophils, 5% bands, 2% metamyelocytes, 2% myelocytes, 6% myeloblasts, 33% lymphocytes, 35%
monocytes, and 2% eosinophils. A bone marrow biopsy specimen shows 90% cellularity with many immature cells,
including ringed sideroblasts, megaloblasts, hypolobated megakaryocytes, and myeloblasts. Karyotypic analysis shows 5q
deletions in many cells. Which of the following is most likely now to have occurred in this patient?
□ (A) Relapse of his previous lymphoma
□ (B) Transformation of lymphoma into myeloid leukemia
□ (C) Myelodysplasia related to therapy for the previous tumor
□ (D) De novo acute myeloblastic leukemia
□ (E) Myeloid metaplasia with myelofibrosis

A

(C) This patient has developed a myelodysplasia, characterized by a cellular marrow in which there are maturation

defects in multiple lineages. This diagnosis is supported by the presence of ringed sideroblasts, megaloblasts, abnormal

megakaryocytes, and myeloblasts in the marrow. Because the hematopoietic cells fail to mature normally, they are not

released into the peripheral blood. The patient has pancytopenia and is susceptible to infections. Myelodysplasias are

clonal stem cell disorders that develop de novo or after chemotherapy with alkylating agents, as in this case. The

presence of chromosomal deletions, such as 5q, is a marker of post-therapy myelodysplasia. The morphologic

abnormalities in the marrow are not seen in any of the other listed conditions.

How well did you know this?
1
Not at all
2
3
4
5
Perfectly
9
Q

A 63-year-old woman experiences a burning sensation in her hands and feet. Two months ago, she had an episode of
swelling with tenderness in the right leg, followed by dyspnea and right-sided chest pain. On physical examination, the
spleen and liver now appear to be enlarged. CBC shows hemoglobin, 13.3 g/dL; hematocrit, 40.1%; MCV, 91 μm3; platelet
count, 657,000/mm3; and WBC count, 17,400/mm3. The peripheral blood smear shows abnormally large platelets. Which
of the following is the most likely diagnosis?
□ (A) Essential thrombocythemia
□ (B) Chronic myelogenous leukemia
□ (C) Myelofibrosis with myeloid metaplasia
□ (D) Acute myelogenous leukemia
□ (E) Polycythemia vera

A

A) Essential thrombocythemia is a myeloproliferative disorder. As with all myeloproliferative diseases, the

transformation occurs in a myeloid stem cell. In this form of myeloproliferative disease, the dominant cell type affected is

the megakaryocyte, and there is thrombocytosis. Other myeloproliferative disorders, such as chronic myelogenous

leukemia, myelofibrosis, and polycythemia vera, also can be accompanied by an increased platelet count. The diagnosis

of essential thrombocytosis can be made after other causes of reactive thrombocytosis are excluded, and if the bone

marrow examination shows increased megakaryocytes with no evidence of leukemia. The throbbing, burning pain in the

extremities is caused by platelet aggregates that occlude small arterioles. The major manifestation of this disease is

thrombotic or hemorrhagic crises. The swelling in this patient’s leg represents phlebothrombosis, followed by pulmonary

embolism with infarction. The peripheral blood WBC count would be high in acute myelogenous leukemia, without

thrombocytosis.

How well did you know this?
1
Not at all
2
3
4
5
Perfectly
10
Q

A 9-year-old boy living in Uganda has had increasing pain and swelling on the right side of his face over the past 8
months. On physical examination, there is a large, nontender mass involving the mandible, which deforms the right side of
his face. There is no lymphadenopathy and no splenomegaly, and he is afebrile. A biopsy of the mass is performed.
Microscopically, the specimen is composed of intermediate-sized lymphocytes with a high mitotic rate. A chromosome
analysis shows a 46,XY,t(8;14) karyotype in these cells. The hemoglobin concentration is 13.2 g/dL, platelet count is
272,000/mm3, and WBC count is 5820/mm3. Infection with which of the following is most likely to be causally related to the
development of these findings?
□ (A) Cytomegalovirus
□ (B) Epstein-Barr virus
Robbins & Cotran Review of Pathology Pg. 244
□ (C) Hepatitis B virus
□ (D) HIV
□ (E) Human papillomavirus
□ (F) Respiratory syncytial virus

A

(B) This patient has the endemic African variety of Burkitt lymphoma, a B-cell lymphoma that typically appears in the

maxilla or mandible of the jaw. This particular neoplasm is related to Epstein-Barr virus infection. Cytomegalovirus

infection occurs in immunocompromised patients and can be a congenital infection, but it is not a direct cause of

neoplasia. Hepatitis B virus infection can be a risk factor for hepatocellular carcinoma. HIV infection can be a risk factor for

the development of non-Hodgkin lymphomas, but most of these are either diffuse large B-cell lymphomas or small

noncleaved Burkitt-like lymphomas. Human papillomavirus infection is related to the formation of squamous dysplasias and

carcinomas, most commonly those involving the cervix. Respiratory syncytial virus infection produces pneumonia in infants

and young children, but is not related to development of neoplasms

How well did you know this?
1
Not at all
2
3
4
5
Perfectly
11
Q

A 23-year-old man undergoing chemotherapy for acute lymphoblastic leukemia has developed a fever and abdominal
pain within the past week. He now has a severe cough. On physical examination, his temperature is 38.4°C. On
auscultation, crackles are heard over all lung fields. Laboratory studies show hemoglobin, 12.8 g/dL; hematocrit, 39%;
MCV, 90 μm3; platelet count, 221,000/mm3; and WBC count, 16,475/mm3 with 51% segmented neutrophils, 5% bands,
18% lymphocytes, 8% monocytes, and 18% eosinophils. Infection with which of the following organisms is most likely to be
complicating the course of this patient’s disease?
□ (A) Cryptococcus neoformans
□ (B) Cytomegalovirus
□ (C) Helicobacter pylori
□ (D) Hepatitis C virus
□ (E) Pseudomonas aeruginosa
□ (F) Strongyloides stercoralis
□ (G) Toxoplasma gondii
□ (H) Varicella-zoster virus

A

(F) The eosinophilia suggests a parasitic infestation. Immunocompromised individuals can have superinfection and

dissemination with strongyloidiasis. The other organisms listed are not known to be associated with eosinophilia

How well did you know this?
1
Not at all
2
3
4
5
Perfectly
12
Q

A 61-year-old man reports a history of back pain for 5 months. He has recently developed a cough that is productive of
yellow sputum. On physical examination, he is febrile, and diffuse rales are heard on auscultation of the lungs. He has no
lymphadenopathy or splenomegaly. Laboratory studies include a sputum culture that grew Streptococcus pneumoniae.
The serum creatinine level is 3.7 mg/dL, and the urea nitrogen level is 35 mg/dL. The figure shows a skull radiograph.
During his hospitalization, a bone marrow biopsy is performed. Which of the following is the biopsy specimen most likely to
show?
□ (A) Scattered small granulomas
□ (B) Numerous plasma cells
□ (C) Nodules of small mature lymphocytes
□ (D) Occasional Reed-Sternberg cells
□ (E) Hypercellularity with many blasts

A

(B) Multiple myeloma produces mass lesions of plasma cells in bone that lead to lysis and pain. The skull radiograph

shows typical punched-out lytic lesions, produced by expanding masses of plasma cells. The Ig genes in myeloma cells

always show evidence of somatic hypermutation. Bence Jones proteinuria can damage the tubules and give rise to renal

failure. Multiple myeloma can be complicated by AL amyloid, which also can lead to renal failure. Patients with myeloma

often have infections with encapsulated bacteria because of decreased production of IgG, required for opsonization.

Granulomatous disease (which is not produced by pneumococcus) can involve the marrow, but usually it does not produce

such sharply demarcated lytic lesions. Nodules of small lymphocytes suggest a small-cell lymphocytic leukemia/lymphoma,

which is not likely to produce lytic lesions. Reed-Sternberg cells suggest Hodgkin lymphoma. Blasts suggest a leukemic

process

How well did you know this?
1
Not at all
2
3
4
5
Perfectly
13
Q

A 26-year-old man has noted lumps in his neck that have been enlarging for the past 6 months. On physical
examination, he has a group of enlarged, nontender right cervical lymph nodes. A biopsy of one of the lymph nodes shows
Robbins & Cotran Review of Pathology Pg. 245
scattered Reed-Sternberg cells, macrophages, lymphocytes, neutrophils, eosinophils, and a few plasma cells. Which of
the following factors elaborated by the Reed-Sternberg cells has led to the appearance of the eosinophils within this
lesion?
□ (A) Platelet-derived growth factor
□ (B) Cyclin D1
□ (C) Interleukin-5
□ (D) Trans-retinoic acid
□ (E) Erythropoietin

A

(C) Interleukin-5 acts as an eosinophilic chemotactic factor to form an eosinophilic cellular component of the mixed

cellularity and nodular sclerosis types of Hodgkin lymphoma. In contrast, transforming growth factor-β secreted by eosinophils promotes the fibrosis that is part of nodular sclerosing Hodgkin lymphoma. Platelet-derived growth factor does

not play a major role in Hodgkin lymphoma, although it may be elaborated by cells in some carcinomas and gliomas. Cyclin

D1 is involved in the cell cycle and proliferation. Trans-retinoic acid is used in treating acute promyelocytic leukemia, in

which the abnormal gene fusion product of the t(15;17) blocks myeloid maturation at the promyelocyte stage.

Erythropoietin drives erythroid cell line proliferation.

How well did you know this?
1
Not at all
2
3
4
5
Perfectly
14
Q

A 53-year-old woman has experienced nausea with vomiting and early satiety for the past 7 months. On physical
examination, she is afebrile and has no lymphadenopathy or hepatosplenomegaly. CBC shows hemoglobin, 12.9 g/dL;
hematocrit, 41.9%; platelet count, 263,000/mm3; and WBC count, 8430/mm3. An upper gastrointestinal endoscopy shows
loss of the rugal folds of the stomach over a 4 × 8 cm area of the fundus. Gastric biopsy specimens reveal the presence of
Helicobacter pylori organisms in the mucus overlying superficial epithelial cells. There are mucosal and submucosal
monomorphous infiltrates of small lymphocytes, which are CD19+ and CD20+, but CD3−. After treatment of the H. pylori
infection, her condition improves. What is the most likely diagnosis?
□ (A) Acute lymphoblastic leukemia
□ (B) Chronic lymphocytic leukemia
□ (C) Diffuse large B-cell lymphoma
□ (D) Follicular lymphoma
□ (E) Hodgkin lymphoma, mixed cellularity type
□ (F) MALT (marginal zone) lymphoma
□ (G) Waldenström macroglobulinemia

A

(F) These lymphomas arise in middle-aged adults at sites of autoimmune or infectious stimulation. If the lesion is

associated with lymphoid tissue, it is sometimes called a mucosa-associated lymphoid tissue tumor (MALT lymphoma, or

MALToma). The most common sites are the thyroid (in Hashimoto thyroiditis), the salivary glands (in Sjögren syndrome),

or the stomach (in Helicobacter pylori infection). Although monoclonal (similar to a neoplasm), these MALT lesions can

regress with antibiotic therapy for H. pylori. A MALT lesion can transform to diffuse large B-cell lymphoma. The cells

correspond to the marginal B-cells found at the periphery of stimulated lymphoid follicles. The other conditions listed are

neoplastic conditions that are not related to H. pylori, and that require chemotherapy to control.

How well did you know this?
1
Not at all
2
3
4
5
Perfectly
15
Q

An 18-month-old girl has developed seborrheic skin eruptions over the past 3 months. She has had recurrent upper
respiratory and middle ear infections with Streptococcus pneumoniae for the past year. Physical examination indicates
that she also has hepatosplenomegaly and generalized lymphadenopathy. Her hearing is reduced in the right ear. A skull
radiograph shows an expansile, 2-cm lytic lesion involving the right temporal bone. Laboratory studies show no anemia,
thrombocytopenia, or leukopenia. The mass is curetted. Which of the following is most likely to be seen on microscopic
examination of this mass?
□ (A) Histiocytes with Birbeck granules
□ (B) Lymphoblasts
□ (C) Plasma cells with Russell bodies
□ (D) Reed-Sternberg cells
□ (E) Ringed sideroblasts
□ (F) Sézary cells

A

The child has Letterer-Siwe disease, a form of Langerhans cell histiocytosis. The Birbeck granules are a distinctive

feature, identified by electron microscopy, which are found in the cytoplasm of the Langerhans cells. Lymphoblasts that

mark as T cells are seen in anterior mediastinal (thymic) masses in children with acute lymphoblastic leukemia/lymphoma.

Plasma cells are seen in multiple myeloma, a disease of older adults accompanied by a monoclonal gammopathy. Reed-

Sternberg cells are seen in Hodgkin lymphoma, which is an unlikely disease in children. Ringed sideroblasts can be seen

in myelodysplastic syndromes. Sézary cells can be seen in peripheral T-cell lymphoma/leukemias, which often involve the

skin.

How well did you know this?
1
Not at all
2
3
4
5
Perfectly
16
Q

A 20-year-old man is left at the door of the emergency department by his “friends” after they spent an evening at a local
pub. On examination, his vital signs are temperature, 37°C; pulse, 110/min; respirations, 20/min; and blood pressure,
75/40 mm Hg. He has left upper quadrant tenderness on palpation. An abdominal CT scan was obtained and is shown in
the figure. What is the most likely etiology for this man’s findings?
□ (A) Amyloidosis
□ (B) Cirrhosis
□ (C) Gaucher disease
□ (D) Myeloproliferative disorder
□ (E) Nonbacterial thrombotic endocarditis
□ (F) Salmonella typhi infection
□ (G) Trauma

A

(G) This patient has a splenic rupture with hematoma formation in a normal-sized spleen. The likelihood of acute

alcohol ingestion favors trauma from falls, fights, and vehicular accidents. A “spontaneous” rupture of the spleen in the

absence of trauma may occur when there is splenic enlargement from infections and neoplasms. Other causes of splenic

enlargement include amyloid deposition, congestive splenomegaly from portal hypertension with cirrhosis, storage

diseases such as Gaucher disease, and myeloproliferative disorders. Embolic events from endocarditis are most likely to

occur with infective endocarditis and nonbacterial thrombotic endocarditis

How well did you know this?
1
Not at all
2
3
4
5
Perfectly
17
Q

A 34-year-old man has experienced multiple nosebleeds along with bleeding gums for the past month. On examination,
his temperature is 37.3°C. He has multiple cutaneous ecchymoses. Laboratory studies show hemoglobin, 8.5 g/dL;
hematocrit, 25.7%; platelet count, 13,000/mm3; and WBC count, 52,100/mm3 with 5% segmented neutrophils, 5% bands,
2% myelocytes, 83% blasts, 3% lymphocytes, and 2% monocytes. Examination of his peripheral blood smear shows the
blasts have delicate nuclear chromatin along with fine cytoplasmic azurophilic granules. These blasts are CD33+. Which
of the following morphologic findings is most likely to be present on his peripheral blood smear?
□ (A) Auer rods
□ (B) Döhle bodies
□ (C) Hairy projections
□ (D) Heinz bodies
□ (E) Sickle cells
□ (F) Toxic granulations

A

(A) This patient has acute myelogenous leukemia, which infiltrates the marrow and reduces normal hematopoiesis to

account for anemia and marked thrombocytopenia. The initial presentation may be acute. The Auer rods are

condensations of the azurophilic granules. Döhle bodies, which are patches of dilated endoplasmic reticulum, and toxic

granulations, which are coarse and dark primary granules, are reactive changes in mature neutrophils most indicative of

marked inflammation, such as bacterial sepsis. Hairy projections are seen on the circulating B cells of hairy-cell leukemia.

Heinz bodies are seen in G6PD deficiency and are precipitates of denatured globin. Sickling of RBCs is a feature of sickle

cell anemia, which is not related to leukemia

How well did you know this?
1
Not at all
2
3
4
5
Perfectly
18
Q

A 41-year-old man has had fevers with chills and rigors for the past 2 weeks. On physical examination, his temperature
is 39.2°C. CBC shows hemoglobin, 13.9 g/dL; hematocrit, 40.5%; MCV, 93 μm3; platelet count, 210,000/mm3; and WBC
count, 13,750/mm3 with 75% segmented neutrophils, 10% bands, 10% lymphocytes, and 5% monocytes. A bone marrow
biopsy specimen shows hypercellularity with a marked increase in myeloid precursors at all stages of maturation and in
band neutrophils. These findings are most likely caused by which of the following conditions?
□ (A) Acute viral hepatitis
□ (B) Glucocorticoid therapy
□ (C) Lung abscess
□ (D) Vigorous exercise
□ (E) Acute myelogenous leukemia

A

(C) Chronic infections and chronic inflammatory conditions, such as lung abscesses, can lead to an expansion of the

myeloid precursor pool in the bone marrow; this manifests as neutrophilic leukocytosis. Acute viral hepatitis, in contrast to

acute bacterial infections, does not cause neutrophilic leukocytosis. Glucocorticoids can increase the release of marrow

storage pool cells and diminish extravasation of neutrophils into tissues. Vigorous exercise can produce neutrophilia

transiently from demargination of neutrophils. In acute myelogenous leukemia, the marrow is filled with blasts, not maturing

myeloid elements.

How well did you know this?
1
Not at all
2
3
4
5
Perfectly
19
Q

A 37-year-old man known to have been infected with HIV for the past 10 years is admitted to the hospital with
abdominal pain of 3 days’ duration. Physical examination shows abdominal distention and absent bowel sounds. An
abdominal CT scan shows a mass lesion involving the ileum. He undergoes surgery to remove an area of bowel
obstruction in the ileum. Gross examination of the specimen shows a firm, white mass, 10 cm long and 3 cm at its greatest
depth. The mass has infiltrated through the wall of the ileum. Histologic studies show a mitotically active population of
CD19+ lymphoid cells with prominent nuclei and nucleoli. Molecular analysis is most likely to show which of the following
viral genomes in the lymphoid cells?
□ (A) Epstein-Barr virus
□ (B) HIV
□ (C) Human herpesvirus type 8
□ (D) Human T-cell leukemia/lymphoma virus type 1
□ (E) Cytomegalovirus

A

(A) This HIV-positive patient has an extranodal infiltrative mass, composed of B cells (CD19+), in the ileum. This is a

diffuse large cell lymphoma of B cells. These tumors contain the Epstein-Barr virus (EBV) genome, and it is thought that

immunosuppression allows unregulated proliferation and neoplastic transformation of EBV-infected B cells. HIV is not seen

in normal or neoplastic B cells. Human herpesvirus type 8 (also called Kaposi sarcoma herpesvirus) is found in the spindle

cells of Kaposi sarcoma and in body cavity B-cell lymphomas in patients with AIDS. Human T-cell leukemia/lymphoma

Robbins & Cotran Review of Pathology Pg. 263

virus type 1 is related to HIV-1, and it causes adult T-cell leukemia/lymphoma. Cytomegalovirus is not known to cause any

tumors

How well did you know this?
1
Not at all
2
3
4
5
Perfectly
20
Q

A 70-year-old man has experienced increasing fatigue for the past 6 months. On physical examination, he has
nontender axillary and cervical lymphadenopathy, but there is no hepatosplenomegaly. The hematologic work-up shows
hemoglobin, 9.5 g/dL; hematocrit, 28%; MCV, 90 μm3; platelet count, 120,000/mm3; and WBC count, 42,000/mm3. The
peripheral blood smear shows a monotonous population of small, round, mature-looking lymphocytes. Flow cytometry
shows these cells to be CD19+, CD5+, and TdT−. Which of the following is most likely to be seen with cytogenetic and
molecular analysis of the cells in the patient’s blood?
□ (A) t(9;22) leading to BCR-ABL rearrangement
□ (B) Clonal rearrangement of immunoglobulin genes
□ (C) Clonal rearrangement of T-cell receptor genes
□ (D) t(8;14) leading to C-MYC overexpression
□ (E) t(14;18) leading to BCL2 overexpression

A

(B) The clinical history, the peripheral blood smear, and the phenotypic markers are characteristic of chronic

lymphocytic leukemia, a clonal B-cell neoplasm in which immunoglobulin genes are rearranged, and T-cell receptor genes

are in germline configuration. The t(9;22) is a feature of chronic myeloid leukemia. The t(8;14) translocation is typical of

Burkitt lymphoma; this lymphoma occurs in children at extranodal sites. The t(14;18) translocation is a feature of follicular

lymphomas, which are distinctive B-cell tumors that involve the nodes and produce a follicular pattern. The lymphoma cells

can be present in blood, but they do not look like mature lymphocytes

How well did you know this?
1
Not at all
2
3
4
5
Perfectly
21
Q

A 69-year-old woman complains of increasing back pain for 1 month. On physical examination, there is tenderness over
the lower back, but no kyphosis or scoliosis. A radiograph of the spine shows a partial collapse of T11 and several 0.5- to
1.5-cm lytic lesions with a rounded “soap-bubble” appearance in the thoracic and lumbar vertebrae. A bone marrow biopsy
is performed, and a smear of the aspirate is shown in the figure. Which of the following is the most likely laboratory finding
in this patient?
□ (A) Bence Jones proteins in the urine
□ (B) t(9;22) in the karyotype of marrow
□ (C) Elevated leukocyte alkaline phosphatase score
□ (D) Decreased serum alkaline phosphatase level
□ (E) Platelet count of 750,000/mm3
□ (F) WBC count of 394,000/mm3

A

(A) The characteristic “punched-out” bone lesions of multiple myeloma seen on radiographs result from bone

destruction mediated by RANKL, a cytokine produced by the myeloma cells that activates osteoclasts. Several cytokines,

most notably IL-6, are important growth factors for plasma cells. They are produced by tumor cells and resident marrow

stromal cells. High serum levels of IL-6 correlate with active disease and poor prognosis. The monoclonal population of

plasma cells often produces a monoclonal serum “spike” seen in serum or urine protein electrophoresis. Patients can have

hypercalcemia and an increased serum alkaline phosphatase level. The neoplastic cells are generally well differentiated,

with features such as a perinuclear hof, similar to normal plasma cells. The t(9;22) translocation is the Philadelphia

chromosome seen in chronic myelogenous leukemia (CML). CML and other myeloproliferative disorders sometimes are

accompanied by a thrombocytosis, but are unlikely to produce mass lesions or bony destruction. Leukemias also can fill

the marrow space, but generally do not destroy bone.

How well did you know this?
1
Not at all
2
3
4
5
Perfectly
22
Q

A 33-year-old woman reports having generalized fatigue and night sweats for 3 months. Physical examination shows
nontender right cervical lymphadenopathy. Biopsy of one lymph node shows a microscopic pattern of thick bands of
fibrous connective tissue with intervening lymphocytes, plasma cells, eosinophils, macrophages, and occasional Reed-
Sternberg cells. An abdominal CT scan and bone marrow biopsy specimen show no abnormalities. Which of the following
is the most likely subtype and stage of this patient’s disease?
□ (A) Lymphocyte predominance, stage I
□ (B) Lymphocyte predominance, stage II
□ (C) Nodular sclerosis, stage I
□ (D) Mixed cellularity, stage II
□ (E) Lymphocyte depletion, stage III

A

C) The bands of fibrosis are typical of the nodular sclerosis type of Hodgkin lymphoma, which is most commonly seen

in young adults, particularly women. Involvement of one group of lymph nodes places this in stage I. Mediastinal

involvement is common. Most of such cases are stage I or II, and the prognosis of such early-stage cases is good

How well did you know this?
1
Not at all
2
3
4
5
Perfectly
23
Q

A 7-year-old boy has complained of worsening pain in the right side of his groin region for the past week. Physical
examination shows painful, swollen lymph nodes in the right inguinal region. An inguinal lymph node biopsy is performed.
Histologically, the node has large, variably sized, germinal centers containing tingible-body macrophages and numerous
mitotic figures. There are numerous parafollicular and sinusoidal neutrophils. What is the most likely cause of these
histologic changes?
□ (A) Acute lymphoblastic leukemia
□ (B) Sarcoidosis
□ (C) Follicular lymphoma
□ (D) Cat-scratch disease
□ (E) Acute lymphadenitis
□ (F) Toxoplasmosis

A

(E) Painful and acute enlarged nodes suggest a reactive condition and not a neoplastic process, such as a lymphoma

or a leukemia. In children, enlarged tender nodes and acute lymphadenitis are common. Many infectious processes can

give rise to these findings, particularly bacterial infections. Children are quite active and acquire plenty of cuts and scrapes

on extremities, which can become infected. Sarcoidosis is a chronic granulomatous process typically seen in adults and

characterized by the formation of noncaseating granulomas. Follicular lymphomas are B-cell neoplasms that efface the

normal architecture of the lymph nodes; these tumors do not occur in children. Cat-scratch disease can produce sarcoidlike

granulomas with stellate abscesses. Toxoplasmosis can be a congenital infection or can be seen in

immunocompromised individuals; it produces a pattern of follicular hyperplasia.

How well did you know this?
1
Not at all
2
3
4
5
Perfectly
24
Q

A 15-year-old boy has developed a cough and a high fever over the past 4 days. On physical examination, he has a
temperature of 39.2°C. Diffuse rales are heard over all lung fields. Laboratory studies show hemoglobin, 14.8 g/dL;
hematocrit, 44.4%; platelet count, 496,000/mm3; and WBC count, 15,600/mm3. Examination of the peripheral blood smear
shows RBCs with marked anisocytosis and Howell-Jolly bodies. A sputum culture grows Haemophilus influenzae. Which
of the following is the most likely diagnosis?
□ (A) DiGeorge syndrome
□ (B) Galactosemia
□ (C) Gaucher disease
□ (D) Myeloproliferative disorder
□ (E) Prior splenectomy
□ (F) Trisomy 21

A

(E) Splenectomy in childhood reduces immunity to encapsulated bacterial organisms. The spleen recycles old RBCs

and removes inclusions such as Howell-Jolly bodies (similar to getting the cherry pits out without damaging the cherry).

About one third of all circulating platelets are pooled in the spleen, and granulocytes are marginated in splenic sinusoids,

so that when the spleen is absent, the WBC and platelet counts increase. DiGeorge syndrome leads to cell-mediated

immunodeficiency and increased viral, fungal, and parasitic diseases. Galactosemia results from an inborn error of

metabolism, leading to liver disease and fibrosis that can cause splenomegaly. Gaucher disease leads to splenomegaly

without significant immunodeficiency. Myeloproliferative disorders increase the size of the spleen. The thymus, but not the

spleen, is sometimes involved in patients with Down syndrome (trisomy 21).

25
Q

A 4-year-old boy has appeared listless for about 1 week. He now complains of pain when he is picked up by his
mother, and he exhibits irritability when his arms or legs are touched. In the past 2 days, several large ecchymoses have
appeared on the right thigh and left shoulder. CBC shows hemoglobin, 10.2 g/dL; hematocrit, 30.5%; MCV, 96 μm3;
platelet count, 45,000/mm3; and WBC count, 13,990/mm3. Examination of the peripheral blood smear shows blasts that
lack peroxidase-positive granules, but contain PAS-positive aggregates and stain positively for TdT. Flow cytometry
shows the phenotype of blasts to be CD19+, CD3−, and sIg−. Which of the following is the most likely diagnosis?
□ (A) Chronic myelogenous leukemia
□ (B) Idiopathic thrombocytopenic purpura
□ (C) Acute myelogenous leukemia
□ (D) Chronic lymphocytic leukemia
□ (E) Acute lymphoblastic leukemia

A

(E) These findings are characteristic of a childhood acute lymphoblastic leukemia of the precursor-B cell type. The

rapid expansion of the marrow caused by proliferation of blasts can lead to bone pain and tenderness. Features

supporting an acute leukemia are anemia, thrombocytopenia, and the presence of blasts in the peripheral blood and bone

marrow. Anemia and thrombocytopenia result from suppression of normal hematopoiesis by the leukemic clone in the
marrow. The phenotype of CD19+, CD3−, and sIg− is typical of pre-B cells. TdT is a marker of early T cell–type and B

cell–type lymphoid cells. Chronic myelogenous leukemia is a disease of adults, and the WBC count is quite high; the peripheral blood contains some myeloblasts, but other stages of myeloid differentiation also are detected. In idiopathic

thrombocytopenic purpura, only the platelet count is reduced because of antibody-mediated destruction of platelets. An

acute myelogenous leukemia is a disease of young to middle-aged adults, and there would be peroxidase-positive

myeloblasts and phenotypic features of myeloid cells. Chronic lymphocytic leukemia is a disease of older adults; patients

have many small circulating mature B lymphocytes.

26
Q

A 49-year-old woman has experienced increasing weakness and chest pain over the past 6 months. On physical
examination, she is afebrile and normotensive. Motor strength is 5/5 in all extremities, but diminishes to 4/5 with repetitive
movement. There is no muscle pain or tenderness. Laboratory studies show hemoglobin, 14 g/dL; hematocrit, 42%;
platelet count, 246,000/mm3; and WBC count, 6480/mm3. A chest CT scan shows an irregular 10 × 12 cm anterior
mediastinal mass. The surgeon has difficulty removing the mass because it infiltrates surrounding structures.
Microscopically, the mass is composed of large, spindled, atypical epithelial cells mixed with lymphoid cells. Which of the
following is the most likely diagnosis of this lesion?
□ (A) Granulomatous inflammation
□ (B) Hodgkin lymphoma
□ (C) Lymphoblastic lymphoma
□ (D) Malignant thymoma
□ (E) Metastatic breast carcinoma
□ (F) Organizing abscess

A

(D) Thymomas are rare neoplasms that can be benign or malignant. In one third to one half of cases, thymomas

produce myasthenia gravis as an initial presentation (as in this case). Benign thymomas have a mixed population of

lymphocytes and epithelial cells and are circumscribed, whereas malignant thymomas are invasive and have atypical cells.

Thymic carcinomas resemble squamous cell carcinomas. Granulomas can have epithelioid macrophages and

lymphocytes, but the thymus is an unusual location. Hodgkin lymphoma involves lymph nodes in the middle or posterior

mediastinum. Lymphoblastic lymphoma of the T cell variety is seen in the thymus in children, and it has no epithelial
component. Metastases to the thymus are quite unusual. An organizing abscess could have granulation tissue at its edge,

with a mixture of inflammatory cell types, but not atypical cells.

27
Q

A 69-year-old man notices the presence of “lumps” in the right side of his neck that have been enlarging over the past
year. Physical examination shows firm, nontender posterior cervical lymph nodes 1 to 2 cm in diameter. The overlying skin
is intact and not erythematous. A lymph node is biopsied. Which of the following histologic features provides the best
evidence for malignant lymphoma in this node?
□ (A) Presence of lymphoid cells positive for kappa, but not lambda, light chains
□ (B) Absence of a pattern of follicles with germinal centers
□ (C) Proliferation of small capillaries in the medullary and paracortical regions
□ (D) Presence of cells that stain with monoclonal antibody to the CD30 antigen
□ (E) Absence of plasma cells and immunoblasts in sinusoidal spaces

A

(A) All lymphoid neoplasms are derived from a single transformed cell and are monoclonal. Monoclonality in B-cell

neoplasms, which comprise 80% to 85% of all lymphoid neoplasms, often can be shown by staining for light chains.

Populations of normal or reactive (polyclonal) B cells contain a mixture of B cells expressing kappa and lambda light

chains. Some lymphoid neoplasms have a follicular pattern. A normal pattern of follicles is sometimes absent if the node is

involved, as in some inflammatory conditions or in immunosuppression. A proliferation of capillaries is typically a benign,

reactive process. The CD30 antigen is a marker for activated T and B cells. Plasma cells are variably present in reactive

conditions, but their absence does not indicate malignancy.

28
Q

A 62-year-old man visits his physician because of prolonged fever and a 4-kg weight loss over the past 6 months. On
physical examination, his temperature is 38.6°C. He has generalized nontender lymphadenopathy, and the spleen tip is
palpable. Laboratory studies show hemoglobin, 10.1 g/dL; hematocrit, 30.3%; platelet count, 140,000/mm3; and WBC
count, 24,500/mm3 with 10% segmented neutrophils, 1% bands, 86% lymphocytes, and 3% monocytes. A cervical lymph
node biopsy specimen shows a nodular pattern of small lymphoid cells. A bone marrow specimen shows infiltrates of
similar small cells having surface immunoglobulin that are CD5+, but CD10−. Cytogenetic analysis indicates t(11;14) in
these cells. What is the most likely diagnosis?
□ (A) Mantle cell lymphoma
□ (B) Follicular lymphoma
□ (C) Acute lymphoblastic leukemia
□ (D) Burkitt lymphoma
□ (E) Small lymphocytic lymphoma

A

(A) The immunophenotype is characteristic for mantle cell lymphoma. Of the lesions listed, lymphoblastic lymphoma

and Burkitt lymphoma occur in a much younger age group. Burkitt lymphoma has a t(8;14) translocation. The remaining

three lesions occur in an older age group. Of these, small lymphocytic lymphoma manifests with absolute lymphocytosis

and the peripheral blood picture of chronic lymphocytic leukemia. Follicular lymphoma has a distinct and characteristic

translocation t(14;18) involving the BCL2 gene. In contrast, mantle cell lymphoma, seen in older men, has the t(11;14)

translocation, which activates the cyclin D1 (BCL1) gene; these tumors do not respond well to chemotherapy

29
Q

A 45-year-old man has experienced recurrent fevers and a 6-kg weight loss over the past 5 months. On physical
examination, his temperature is 37.5°C, and he has cervical lymphadenopathy. The patient reports that the adenopathy
becomes very tender after he drinks a six-pack of beer. A lymph node biopsy specimen shows effacement of the nodal
architecture by a population of small lymphocytes, plasma cells, eosinophils, and macrophages. Which of the following additional cell types, which stains positively for CD15, is most likely to be found in this disease?
□ (A) Reed-Sternberg cell
□ (B) Immunoblast
□ (C) Epithelioid cell
□ (D) Neutrophils
□ (E) Mast cell

A

(A) The features suggest Hodgkin lymphoma (HL), mixed cellularity type, which tends to affect older men. As in all

other forms of HL but lympocyte predominance, the Reed-Sternberg cells and variants stain with CD15. These cells also

express CD30, an activation marker on T cells, B cells, and monocytes. Clinical symptoms are common in the mixed

cellularity type of HL, and this histologic type tends to manifest in advanced stages. The pain associated with alcohol

consumption is a paraneoplastic phenomenon peculiar to HL. The Reed-Sternberg cells make up a relatively small

percentage of the tumor mass, with most of the cell population consisting of reactive cells such as lymphocytes, plasma

cells, macrophages, and eosinophils. Immunoblasts suggest a B-cell proliferation. Epithelioid cells are seen in

granulomatous inflammatory reactions. Neutrophils accumulate at sites of acute inflammation. Mast cells are not numerous

in HL; they participate in type I hypersensitivity responses.

30
Q

A 23-year-old woman has noticed that she develops a skin rash if she spends prolonged periods outdoors. She has a
malar skin rash on physical examination. Laboratory studies include a positive ANA test result with a titer of 1 : 1024 and a
“rim” pattern. An anti–double-stranded DNA test result also is positive. The hemoglobin concentration is 12.1 g/dL,
hematocrit is 35.5%, MCV is 89 μm3, platelet count is 109,000/mm3, and WBC count is 4500/mm3. Which of the following
findings is most likely to be shown by a WBC differential count?
□ (A) Eosinophilia
□ (B) Thrombocytosis
□ (C) Monocytosis
□ (D) Neutrophilia
□ (E) Basophilia

A

This patient has evidence of an autoimmune disease, most likely systemic lupus erythematosus (SLE). SLE can be

accompanied by monocytosis. (Cytopenias also can occur in SLE because of autoantibodies against blood elements, a

form of type II hypersensitivity.) Eosinophilia is a feature more often seen in allergic conditions, parasitic infestations, and

chronic myelogenous leukemia (CML). Thrombocytosis usually occurs in neoplastic disorders of myeloid stem cells, such

as the myeloproliferative disorders that include CML. Neutrophilia is seen in acute infectious and inflammatory conditions.

Basophilia occurs infrequently, but also can be seen in CML.

31
Q

A 58-year-old man from Nagasaki, Japan, has noted an increasing number of skin lesions for the past 8 months. On
examination, there are scaling red-brown patches on all skin surfaces. He also has generalized lymphadenopathy and
hepatosplenomegaly. Laboratory studies show hemoglobin, 9.7 g/dL; hematocrit, 31%; MCV, 89 μm3; platelet count,
177,000/mm3; and WBC count, 18,940/mm3 with differential count of 35 segmented neutrophils, 2 band neutrophils, 58
lymphocytes, and 5 monocytes. His serum calcium is 11.5 mg/dL. Examination of his peripheral blood smear shows
multilobated “cloverleaf” cells. Despite aggressive chemotherapy, his condition worsens with development of paresthesias
along with erythematous plaques and red-brown nodules on his skin. Which of the following infectious agents most likely
caused his illness?
□ (A) Cytomegalovirus
□ (B) Epstein-Barr virus (EBV)
□ (C) HIV
□ (D) HTLV-1
□ (E) Rochalimaea henselae
□ (F) Yersinia pestis

A

(D) The patient lives in an area endemic for HTLV-1, which can cause leukemia/lymphoma and demyelinating disease.

The neoplastic lymphoid cells can infiltrate many organs. Skin lesions resemble those of mycosis fungoides. CMV is not associated with development of neoplasms, but it can complicate the course of patients with neoplasms who become

immunocompromised. EBV can be associated with African Burkitt lymphoma. HIV is best known to be associated with

lymphomas and with Kaposi sarcoma, but not leukemias. Cat-scratch disease from R. henselae infection results in

lymphadenopathy with microscopic stellate necrosis. Plague from Y. pestis can produce acute lymphadenopathy with

microscopic hemorrhagic necrosi

32
Q

In an experiment, cell samples are collected from the bone marrow aspirates of patients who were diagnosed with
lymphoproliferative disorders. Cytogenetic analyses are performed on these cells, and a subset of the cases is found to
have the BCR-ABL fusion gene from the reciprocal translocation t(9;22)(q34;11). The presence of this gene results in
increased tyrosine kinase activity. Patients with which of the following conditions are most likely to have this gene?
□ (A) Follicular lymphoma
□ (B) Chronic myelogenous leukemia
□ (C) Hodgkin lymphoma, lymphocyte depletion type
□ (D) Acute promyelocytic leukemia
□ (E) Multiple myeloma

A

(B) This is the Philadelphia chromosome, or Ph1, which is characteristic of patients with chronic myelogenous leukemia

(CML). This karyotypic abnormality can be found using cytogenetic techniques, including fluorescence in situ hybridization

(FISH). In the few cases that appear negative by karyotyping and by FISH, molecular analysis shows BCR-ABL

rearrangements, and the tyrosine kinase activated via this fusion gene is the target of current therapy for CML. This

rearrangement is considered a diagnostic criterion for CML. CML is a disease of pluripotent stem cells that affects all

lineages, but the granulocytic precursors expand preferentially in the chronic phase. Follicular lymphomas have a t(14;18)

karyotypic abnormality involving the BCL2 gene. Hodgkin disease and myelomas usually do not have characteristic

karyotypic abnormalities. Acute promyelocytic leukemias often have the t(15;17) abnormality

33
Q

A 64-year-old man has inguinal, axillary, and cervical lymphadenopathy. The nodes are firm and nontender. A biopsy
specimen of a cervical node shows a histologic pattern of nodular aggregates of small, cleaved lymphoid cells and larger
cells with open nuclear chromatin, several nucleoli, and moderate amounts of cytoplasm. A bone marrow biopsy specimen
shows lymphoid aggregates of similar cells with surface immunoglobulin that are CD10+, but CD5−. Karyotyping of these
lymphoid cells indicates the presence of t(14;18). What is the most likely diagnosis?
□ (A) Hodgkin lymphoma, nodular sclerosis type
□ (B) Acute lymphadenitis
□ (C) Follicular lymphoma
□ (D) Mantle cell lymphoma
□ (E) Toxoplasmosis

A

(C) This patient has follicular lymphoma, the most common form of non-Hodgkin lymphoma among adults in the United

States. Men and women are equally affected. The neoplastic B cells mimic a population of follicular center cells and

produce a nodular or follicular pattern. Nodal involvement is often generalized, but extranodal involvement is uncommon.

The t(14;18) translocation, which is characteristic, causes overexpression of the BCL2 gene; the cells are resistant to

apoptosis. In keeping with this, follicular lymphomas are indolent tumors that continue to accumulate cells for 7 to 9 years.

In Hodgkin lymphoma, there are few Reed-Sternberg cells, surrounded by a reactive lymphoid population. The lymphoid

population in acute lymphadenitis is reactive, and there is no bone marrow involvement. Mantle cell lymphoma also is a Bcell

tumor; it is more aggressive than follicular lymphoma and is typified by the t(11;14) translocation, in which the cyclin

D1 gene (BCL2) is overexpressed. In toxoplasmosis, there would be a mixed population of inflammatory cells and some

necrosis.

34
Q

A 26-year-old man has had a fever with nonproductive cough for the past 10 weeks. On examination, his temperature is
37.4°C. A chest radiograph shows a 4-cm left upper lobe nodule. CBC shows hemoglobin, 13.3 g/dL; hematocrit, 40.5%;
platelet count, 281,000/mm3; and WBC count, 13,760/mm3 with 38% segmented neutrophils, 2% bands, 45% lymphocytes,
and 15% monocytes. What is the most likely diagnosis?
□ (A) Acute lymphoblastic leukemia/lymphoma
□ (B) Hodgkin lymphoma, lymphocyte rich type
□ (C) Mycobacterium tuberculosis granuloma
□ (D) Myelodysplastic syndrome
□ (E) Staphylococcus aureus abscess

A

(C) This patient has granulomatous infection leading to leukocytosis with lymphocytosis and monocytosis. The most

consistent form of leukocytosis from infection is neutrophilia with acute bacterial infections, such as S. aureus infection.

Viral, mycobacterial, and fungal infections produce less consistent peripheral blood findings. An acute lymphoblastic

leukemia is likely to be accompanied by a higher WBC count with circulating lymphoblasts. Hodgkin lymphomas have no

consistent peripheral blood findings and are not likely to produce solitary lung nodules. Myelodysplastic syndromes are

marked by the presence of immature myeloid cells and cytopenias in the peripheral blood

35
Q

A 27-year-old man visits the physician because he has had a fever for the past 10 days. On physical examination, his
temperature is 37.9°C, pulse is 87/min, respirations are 21/min, and blood pressure is 100/55 mm Hg. A diastolic murmur is
heard on auscultation of the chest. The tip of the spleen is palpable and tender. Laboratory studies show hemoglobin,
12.8 g/dL; hematocrit, 38.4%; platelet count, 231,000/mm3; and WBC count, 12,980/mm3 with 70% segmented neutrophils,
6% bands, 1% metamyelocytes, 19% lymphocytes, and 4% monocytes. The representative gross appearance of the
spleen is shown in the figure. What is the most likely diagnosis?
□ (A) Acute myelogenous leukemia
□ (B) Disseminated histoplasmosis
□ (C) Hodgkin lymphoma
□ (D) Infective endocarditis
□ (E) Metastatic carcinoma
□ (F) Micronodular cirrhosis
□ (G) Rheumatic heart disease

A

(D) The pale, tan-to-yellow, firm areas shown in the figure are infarcts. These lesions are either wedge-shaped and

based on the capsule or more irregularly shaped within the parenchyma. Emboli in the systemic arterial circulation can

arise from vegetations on cardiac valves in a patient with infective endocarditis; these can lead to splenic infarction.

Emboli exiting the aorta at the celiac axis generally take the straight route to the spleen. The kidneys and brain are other

common sites for systemic emboli to lodge. Although acute myelogenous leukemia can cause enlargement of the spleen,

there are typically no focal lesions—only uniform infiltration of the parenchyma—but the massive size of the spleen with

chronic myelogenous leukemia predisposes to splenic infarcts. There would be scattered granulomas that are rounded

and tan with granulomatous diseases of the spleen, such as histoplasmosis. In Hodgkin lymphoma, there can be focal

nodules. Metastases can enlarge the spleen, but are uncommon in the spleen and are unlikely to be accompanied by

signs of infection. Similarly, the congestive splenomegaly that occurs in cirrhosis and portal hypertension does not

produce focal splenic lesions. In acute rheumatic fever, the verrucous vegetations are unlikely to embolize; in chronic

rheumatic valvulitis, there is scarring with valve deformity, and this increases the risk of infective endocarditis

36
Q

A 29-year-old, HIV-positive woman has developed fever, cough, and dyspnea over the past week. On physical examination, her temperature is 37.9°C. There is dullness to percussion over lung fields posteriorly. A bronchoalveolar
lavage is performed, and cysts of Pneumocystis carinii are present. She is given trimethoprim/sulfamethoxazole. One
week later, her respiratory status has improved. Laboratory studies now show hemoglobin, 7.4 g/dL; hematocrit, 22.2%;
MCV, 98 μm3; platelet count, 47,000/mm3; and WBC count, 1870/mm3 with 2% segmented neutrophils, 2% bands, 85%
lymphocytes, 10% monocytes, and 1% eosinophils. One week later, she experiences increasing dyspnea, and a chest CT
scan shows multiple 1- to 3-cm nodules with hemorrhagic borders in all lung fields. These nodules are most likely to be
caused by infection with which of the following organisms?
□ (A) Aspergillus fumigatus
□ (B) Bartonella henselae
□ (C) Mycobacterium avium complex
□ (D) Escherichia coli
□ (E) Herpes simplex virus
□ (F) Pneumocystis carinii
□ (G) Toxoplasma gondii

A

(A) This woman developed severe neutropenia with pancytopenia from drug toxicity, which predisposed her to sepsis.

Aspergillosis is a cause of pulmonary nodules, and neutropenia is a significant risk factor. These organisms often invade

blood vessels, producing hemorrhagic lesions. Bartonellosis can produce bacillary angiomatosis, which is more likely to

involve the skin. Mycobacterium avium complex is more likely to involve organs of the mononuclear phagocyte system and unlikely to produce large nodules. Escherichia coli, similar to many bacterial infections, can occur in HIV infection, but it

has a pattern of acute neutrophilic infiltrates. Herpes simplex virus (type 1 or 2) is an unlikely disseminated infection in HIV.

Pneumocystis pneumonia rarely produces nodular lesions. Toxoplasmosis is uncommon in the lung, even in

immunocompromised individuals.

37
Q

A 39-year-old woman felt a lump in her breast 1 week ago. She visits the physician, who palpates a firm, fixed, irregular
3-cm mass in the upper outer quadrant of the right breast and a firm, nontender lymph node in the right axilla. A
lumpectomy and axillary node dissection are performed, and microscopic examination shows an infiltrating ductal
carcinoma present in the breast. Flow cytometric analysis of the node shows a polyclonal population of CD3+, CD19+,
CD20+, and CD68+ cells with no aneuploidy or increase in S phase. Which of the following is most likely to be present on
microscopic examination of this axillary node?
□ (A) Acute lymphadenitis
□ (B) Diffuse large B-cell lymphoma
□ (C) Metastatic infiltrating ductal carcinoma
□ (D) Necrotizing granulomas
□ (E) Plasmacytosis
□ (F) Sinus histiocytosis

A

(F) Lymph nodes draining from a cancer often show a reactive pattern, with dilated sinusoids that have endothelial

hypertrophy and are filled with histiocytes (i.e., macrophages). Sinus histiocytosis represents an immunological response

to cancer antigens. Not all enlarged nodes are caused by metastatic disease in cancer patients. CD3 is a T cell marker,

CD19 and CD20 are B cell markers, and CD68 is a macrophage (histiocyte) marker. Polyclonal proliferations are typically

benign reactive processes, whereas a monoclonal proliferation suggests a neoplasm. Aneuploidy and high S phase are

characteristics of malignant neoplasms; a high S phase mostly occurs in rapidly growing tumors, such as diffuse large Bcell

lymphomas, and in a few carcinomas, such as small-cell anaplastic carcinoma. Inflammation would produce pain and

tenderness, and the patient may be febrile. Generalized inflammatory diseases or chronic infections can increase the

numbers of plasma cells in lymph nodes

38
Q

A 41-year-old man has experienced several bouts of pneumonia over the past year. He now complains of vague
abdominal pain and a dragging sensation. Physical examination shows marked splenomegaly. CBC shows hemoglobin,
8.2 g/dL; hematocrit, 24.6%; MCV, 90 μm3; platelet count, 63,000/mm3; and WBC count, 2400/mm3. The peripheral blood
smear shows many small leukocytes with reniform nuclei and pale blue cytoplasm with threadlike extensions. A chest x-ray
shows patchy infiltrates, and a culture of sputum grows Mycobacterium kansasii. Which of the following laboratory
findings is most characteristic of this disease?
□ (A) CD19, CD20, and CD11c expression in leukocytes
□ (B) Presence of Auer rods in leukocytes
□ (C) Presence of Ph1 chromosome
□ (D) Presence of toxic granulations in neutrophils
□ (E) Monoclonal IgM in serum

A

(A) This patient has hairy cell leukemia, an uncommon neoplastic disorder of B cells. These cells infiltrate the spleen

and marrow. Pancytopenia results from poor production of hematopoietic cells in the marrow and sequestration of the

mature cells in the spleen. There are two characteristic features of this disease: the presence of hairy projections from

neoplastic leukocytes in the peripheral blood smear and coexpression of B cell (CD19, CD20) and monocyte (CD11c)

markers. In the past, staining for tartrate-resistant acid phosphatase was used. Auer rods are seen in myeloblasts in acute

myeloblastic leukemia. The Ph1 chromosome is a distinctive feature of chronic myelogenous leukemia. Toxic granulations

in neutrophils are seen most often in overwhelming bacterial infections. A monoclonal IgM spike is a feature of

lymphoplasmacytic lymphoma (Waldenström macroglobulinemia).

39
Q

A 23-year-old, previously healthy man has experienced malaise and a low-grade fever and sore throat for 2 weeks. On
physical examination, his temperature is 37.6°C, and he has pharyngeal erythema without exudation. There is tender
cervical, axillary, and inguinal lymphadenopathy. Laboratory studies show hemoglobin, 12.2 g/dL; hematocrit, 36.6%;
platelet count, 190,200/mm3; and WBC count, 8940/mm3. The peripheral blood smear is shown in the figure. Which of the
following is the most likely risk factor for the disease that would be diagnosed in this patient?
□ (A) An inherited disorder of globin chain synthesis
□ (B) Transfusion of packed RBCs
□ (C) Close personal contact (kissing) with his date
□ (D) Sharing infected needles for intravenous drug use
□ (E) Ingestion of raw oysters

A

(C) The smear shows large, “atypical” lymphocytes that are present in patients with infectious mononucleosis and

other viral infections, such as those caused by cytomegalovirus. These atypical cells are large lymphocytes with abundant

cytoplasm and a large nucleus with fine chromatin. Infectious mononucleosis is caused by Epstein-Barr virus (EBV) and

transmitted by close personal contact. In patients with infectious mononucleosis, multiple clones of B cells are infected by

EBV. The EBV genes cause proliferation and activation of B cells, and there is polyclonal B cell expansion. These B cells

secrete antibodies with several specificities, including antibodies that cross-react with sheep RBCs. These heterophil

antibodies produce a positive Monospot test result. The atypical lymphocytes are CD8+ T cells that are activated by EBVinfected

B cells. There is no increase in basophils, eosinophils, or monocytes in infectious mononucleosis. Disorders of

globin chain synthesis affect RBCs, as in the thalassemias. Infectious mononucleosis is not known as a transfusionassociated

disease. Likewise, intravenous drug use is typically not a risk factor for infectious mononucleosis, but

individuals sharing infected needles are at risk of bacterial infections, HIV infection, and viral hepatitis. Eating raw oysters

is a risk factor for hepatitis A because oysters that filter polluted seawater concentrate the virus in their tissues.

40
Q

A 38-year-old woman visits her physician because she has had bleeding gums for the past 3 weeks. Physical
examination shows that her gingivae are thickened and friable. She has hepatosplenomegaly and generalized nontender
lymphadenopathy. CBC shows hemoglobin, 11.2 g/dL; hematocrit, 33.9%; MCV, 89 μm 3; platelet count, 95,000/mm3; and
WBC count, 4500/mm3 with 25% segmented neutrophils, 10% bands, 2% metamyelocytes, 55% lymphocytes, 8%
monocytes, and 1 nucleated RBC per 100 WBCs. A bone marrow biopsy specimen shows 100% cellularity, with many
large blasts that are peroxidase negative and nonspecific esterase positive. What is the most likely diagnosis?
□ (A) Acute lymphoblastic leukemia
□ (B) Acute megakaryocytic leukemia
□ (C) Acute promyelocytic leukemia
□ (D) Acute erythroleukemia
□ (E) Acute monocytic leukemia

A

(E) This patient has an “aleukemic” leukemia, in which the peripheral blood count of leukocytes is not high, but the

leukemic blasts fill the marrow. These blasts show features of monoblasts because they are peroxidase negative and

nonspecific esterase positive. This patient has an M5 leukemia, characterized by a high incidence of tissue infiltration and

organomegaly. Acute lymphoblastic leukemia is typically seen in children and young adults. Acute megakaryocytic

leukemia is rare, it is typically accompanied by myelofibrosis, and the blasts react with platelet-specific antibodies. The M3

variant of acute myelogenous leukemia (promyelocytic leukemia) has many promyelocytes filled with azurophilic granules,

making them strongly peroxidase positive. Erythroleukemia is rare and is accompanied by dysplastic erythroid precursors

41
Q

A 7-year-old boy has complained of a severe headache for the past week. On physical examination, there is
tenderness on palpation of long bones, hepatosplenomegaly, and generalized lymphadenopathy. Petechial hemorrhages
are present on the skin. Laboratory studies show hemoglobin, 8.8 g/dL; hematocrit, 26.5%; platelet count, 34,700/mm3;
and WBC count, 14,800/mm3. A bone marrow biopsy specimen shows 100% cellularity, with almost complete replacement
by a population of large cells with scant cytoplasm lacking granules, delicate nuclear chromatin, and rare nucleoli. He
receives a course of chemotherapy and has a complete remission. Which of the following combinations of phenotypic and
karyotypic markers is most likely to be present in this patient?
□ (A) Early pre-B (CD19+, TdT+); hyperdiploidy
□ (B) Early pre-B (CD19+, TdT+); t(9;22)
□ (C) Pre-B (CD5+, TdT+); normal karyotype
□ (D) Pre-B (CD5+, TdT+,); t(9;22)
□ (E) T cell (CD3+, CD2+); normal karyotype
□ (F) T cell (CD3+, CD2+); hyperdiploidy

A

(A) Three markers strongly favor a very good prognosis for acute lymphoblastic leukemia (ALL): early precursor-B cell

type, hyperdiploidy, and patient age between 7 and 10 years, chromosomal trisomy, and t(12;21). Marrow infiltration by the

leukemic cells leads to pancytopenia. Poor prognostic markers for acute lymphoblastic leukemia/lymphoma are T cell

phenotype, patient age younger than 2 years, WBC count >100,000, presence of t(9;22), and presentation in adolescence

and adulthood. In most T cell ALL cases in adolescents, a mediastinal mass arises in the thymus, and lymphoid infiltrates

appear in tissues of the mononuclear phagocyte system.

42
Q

A 51-year-old man visits his physician because the skin of his face, neck, and trunk has become scaly red. He also
complains of intense itching and a 3-kg weight loss over the past 2 months. On physical examination, his temperature is
37.6°C, and he has a generalized exfoliative erythroderma. Generalized nontender lymphadenopathy is present.
Laboratory studies show hemoglobin, 12.9 g/dL; hematocrit, 42%; platelet count, 231,000/mm3; and WBC count,
7940/mm3 with 57% segmented neutrophils, 3% bands, 26% lymphocytes, 5% monocytes, and 9% eosinophils. A skin
biopsy specimen shows the presence of lymphoid cells in the upper dermis and epidermis. These cells have cerebriform
nuclei with marked infolding of nuclear membranes. Similar cells are seen on the peripheral blood smear. Which
combination of the following phenotypic markers is most likely to be expressed on his abnormal lymphocytes?
□ (A) CD3+, CD4+
□ (B) CD5+, CD56+
□ (C) CD10+, CD19+
□ (D) CD13+, CD33+
□ (E) CD19+, sIg+

A

(A) The involvement of skin and the presence of lymphocytes with complex cerebriform nuclei in the skin and the blood

are features of cutaneous T-cell lymphomas. These are malignancies of CD4+ and CD3+ T cells that may produce a

tumor-like infiltration of the skin (mycosis fungoides) or a leukemic picture without tumefaction in the skin (Sézary

syndrome). Cutaneous T-cell lymphomas are indolent tumors, and patients have a median survival of 8 to 9 years. The

other phenotypes provided here are those of CD3−, CD56+ NK cells; mature B cells with CD19+, sIg+;

monocytes/granulocytes with CD33+, CD13+; and neoplastic B cells in chronic lymphocytic leukemia with CD19+, CD5+.

43
Q

A 65-year-old man sees his physician because he has experienced fatigue, a 5-kg weight loss, night sweats, and

abdominal discomfort for the past year. On physical examination, he has marked splenomegaly; there is no

lymphadenopathy. Laboratory studies show hemoglobin, 10.1 g/dL; hematocrit, 30.5%; MCV, 89 μm3; platelet count,

94,000/mm3; and WBC count, 14,750/mm3 with 55% segmented neutrophils, 9% bands, 20% lymphocytes, 8% monocytes,

4% metamyelocytes, 3% myelocytes, 1% eosinophils, and 2 nucleated RBCs per 100 WBCs. The peripheral blood smear

also shows teardrop cells. The serum uric acid level is 12 mg/dL. A bone marrow biopsy specimen shows extensive

marrow fibrosis and clusters of atypical megakaryocytes. Which of the following is most likely to account for the

enlargement in this patient’s spleen?

□ (A) Hodgkin lymphoma

□ (B) Extramedullary hematopoiesis

□ (C) Portal hypertension

□ (D) Granulomas with Histoplasma capsulatum

□ (E) Metastatic adenocarcinoma

A

(B) This patient has classic features of myelofibrosis with myeloid metaplasia. This myeloproliferative disorder also is a

stem cell disorder, in which neoplastic megakaryocytes secrete fibrogenic factors leading to marrow fibrosis. The

neoplastic clone then shifts to the spleen, where it shows trilineage hematopoietic proliferation (extramedullary

hematopoiesis), in which megakaryocytes are prominent. The marrow fibrosis and the extramedullary hematopoiesis in the

spleen fail to regulate orderly release of leukocytes into the blood. The peripheral blood has immature RBC and WBC

precursors (leukoerythroblastic picture). Teardrop RBCs are misshapen RBCs that are seen when marrow undergoes

fibrosis. Marrow injury also can be the result of other causes (e.g., metastatic tumors, irradiation). These causes also can

give rise to a leukoerythroblastic picture, but splenic enlargement with trilineage proliferation usually is not seen. The other

causes mentioned—Hodgkin lymphoma, portal hypertension, and Histoplasma capsulatum infection—can cause splenic

enlargement, but not marrow fibrosis. Adenocarcinoma metastases to the spleen are uncommon.

44
Q

A 60-year-old man has experienced vague abdominal discomfort accompanied by bloating and diarrhea for the past 6

months. On physical examination, there is a mid-abdominal firm mass. The stool is positive for occult blood. An abdominal

CT scan shows a 5 × 12 cm mass involving the wall of the distal ileum and adjacent mesentery. A laparotomy is

performed, and the mass is removed. Microscopically, the mass is composed of sheets of large lymphoid cells with large

nuclei, prominent nucleoli, and frequent mitoses. The neoplastic cells mark with CD19+ and CD20+ and have the BCL6

gene rearrangement. Which of the following prognostic features is most applicable to this case?

□ (A) Indolent, survival of 7 to 9 years without treatment

□ (B) Indolent, can be cured by chemotherapy

□ (C) Indolent, often undergoes spontaneous remission

□ (D) Aggressive, can be cured by chemotherapy

□ (E) Aggressive, often transforms to acute leukemia

□ (F) Aggressive, often spreads to liver, spleen, and marrow

A

(D) This patient has the clinical and morphologic features of diffuse large-cell lymphoma of B cells. These tumors often

involve extranodal sites, show large anaplastic lymphoid cells that involve the tissues diffusely, and contain BCL6 gene

rearrangements. Their clinical course is aggressive, and they become rapidly fatal if untreated. With intensive

chemotherapy, however, 60% to 80% of patients achieve complete remission, and about 50% can be cured. More

aggressive lymphomas tend to be localized, whereas the indolent lymphomas tend to involve multiple nodal sites or

multiple organs such as liver, spleen, and marrow

45
Q

A 45-year-old man has experienced a gradual weight loss and weakness, anorexia, and easy fatigability for 7 months.

Physical examination shows marked splenomegaly. CBC shows hemoglobin, 12.9 g/dL; hematocrit, 38.1%; MCV, 92 μm3;

platelet count, 410,000/mm3; and WBC count, 168,000/mm3. The peripheral blood smear is depicted in part A of the figure.

Karyotypic analysis shows the Ph1 chromosome. The patient undergoes chemotherapy with imatinib mesylate (tyrosine

kinase inhibitor), which reduces the spleen size and brings the total leukocyte count within normal range. He remains in

remission for 3 years and then begins to experience fatigue and a 10-kg weight loss. Physical examination now shows

petechial hemorrhages. CBC shows hemoglobin, 10.5 g/dL; hematocrit, 30%; platelet count, 60,000/μL; and WBC count,

40,000/μL. A peripheral blood smear is depicted in part B of the figure. Karyotypic analysis shows two Ph1 chromosomes

and aneuploidy. Flow cytometric analysis of the peripheral blood shows CD19+, CD10+, sIg−, and CD3− cells. Which of

the following complications of the initial disease did this patient develop after therapy?

□ (A) Sézary syndrome

□ (B) Myelodysplastic syndrome

□ (C) Hairy cell leukemia

□ (D) B lymphoblastic leukemia

□ (E) Acute myeloblastic leukemia

A

(D) This patient came to his physician with a classic history of chronic myelogenous leukemia (CML), confirmed by the

presence of different stages of myeloid differentiation in the blood and by the presence of the Philadelphia chromosome.

He went into a remission and then entered a blast crisis involving B cells (CD19+). The fact that the B cells carry the

original Ph1 chromosome and some additional abnormalities indicates that the B cells and the myeloid cells belong to the

same clone. The best explanation for this is that the initial transforming event affected a pluripotent stem cell, which

differentiated along the myeloid lineage to produce a picture of CML. Analysis, even at this stage, indicates that the

molecular counterpart of the Ph1 chromosome—the BCR-ABL rearrangement—affects all lineages, including B cells, T

cells, and myeloid cells. With the evolution of the disease, additional mutations accumulate in the stem cells, which

differentiate mainly along B lineages, giving rise to B lymphoblastic leukemia; blast crisis also can affect myeloid cells, but

they are not CD19+. The Sézary syndrome has a leukemic component of CD4+ cells in addition to the skin involvement

(mycosis fungoides). Myelodysplastic syndromes can precede the development of acute myelogenous leukemia. Hairy cell

leukemia is an indolent disease without blasts

46
Q

A 14-year-old boy complains of a feeling of discomfort in his chest that has worsened over the past 5 days. On physicalexamination, he ha

s generalized lymphadenopathy. A chest radiograph shows clear lung fields, but there appears to bewidening of the mediastinum. A chest CT scan

shows a 10-cm mass in the anterior mediastinum. A biopsy specimen of themass shows lymphoid cells with lobulated nuclei having delicate, finely

stippled, nuclear chromatin. There is scantcytoplasm, and many mitoses are seen. The cells express TdT, CD2, and CD7 antigens. Molecular analysi

s reveals apoint mutation in the NOTCH1 gene. What is the most likely diagnosis?

□ (A) Lymphoblastic lymphoma

□ (B) Burkitt lymphoma

□ (C) Hodgkin lymphoma, nodular sclerosing type

□ (D) Mantle cell lymphoma

□ (E) Follicular lymphoma

□ (F) Small lymphocytic lymphoma

A

(A) The age and mediastinal location are typical of a lymphoblastic lymphoma involving the thymus. This lesion is

within the spectrum of acute lymphoblastic leukemia or lymphoma (ALL). Most cases of ALL with lymphomatous

presentation are of the pre–T cell type. This fact is supported by the expression of the T-cell markers CD2 and CD7. The

NOTCH1 gene encodes for a transmembrane receptor required for T cell development, and more than half of pre–T cell

tumors have activating point mutations. TdT is a marker of pre-T cells and pre-B cells. A Burkitt lymphoma is a B-cell

lymphoma that also can be seen in adolescents, but usually is present in the jaw or abdomen. Nodular sclerosing Hodgkin

lymphoma does occur in the mediastinum, but it involves mediastinal nodes, not thymus. The histologic features of

Hodgkin lymphoma include the presence of Reed-Sternberg cells, and this variant has fibrous bands intersecting the

lymphoid cells. Mantle cell lymphomas and follicular lymphomas are B-cell tumors usually seen in older patients, and they

do not involve the thymus. Small lymphocytic lymphoma is the tissue phase of chronic lymphocytic leukemia seen in older

adults.

47
Q

A 60-year-old woman has had headaches and dizziness for the past 5 weeks. She has been taking cimetidine for heartburn and omeprazole for ulcers. On physical examination, she is afebrile and normotensive, and her face has a

plethoric to cyanotic appearance. There is mild splenomegaly, but no other abnormal findings. Laboratory studies show

hemoglobin, 21.7 g/dL; hematocrit, 65%; platelet count, 400,000/mm3; and WBC count, 30,000/mm3 with 85%

polymorphonuclear leukocytes, 10% lymphocytes, and 5% monocytes. The peripheral blood smear shows abnormally

Robbins & Cotran Review of Pathology Pg. 256

large platelets and nucleated RBCs. The serum erythropoietin level is undetectable, but the ferritin level is normal. Which

of the following is most characteristic of the natural history of this patient’s disease?

□ (A) Transformation into acute B lymphoblastic leukemia

□ (B) Marrow fibrosis with extramedullary hematopoiesis

□ (C) Spontaneous remissions and relapses without treatment

□ (D) Increase in monoclonal serum immunoglobulin

□ (E) Development of a gastric non-Hodgkin lymphoma

A

(B) This patient has polycythemia vera. The symptoms result from the increased hematocrit and blood volume.

Undetectable erythropoietin in the face of polycythemia is characteristic of polycythemia vera. Polycythemia vera is a

myeloproliferative disorder in which the neoplastic myeloid cells differentiate preferentially along the erythroid lineage.

Other lineages also are affected, however; there is leukocytosis and thrombocytosis. These patients are Ph1 chromosome

negative. Untreated, these patients die of episodes of bleeding or thrombosis—both related to disordered platelet function

and the hemodynamic effects on distended blood vessels. Treatment by phlebotomy reduces the hematocrit. With this

treatment, the disease in 15% to 20% of patients characteristically transforms into myelofibrosis with myeloid metaplasia.

Termination in acute leukemia, in contrast to in chronic myeloid leukemia, is rare. When it occurs, it is an acute myeloid

leukemia, not lymphoblastic leukemia.

48
Q

A clinical study is performed in which the subjects are children 1 to 4 years old who have had multiple infections with

viral, fungal, and parasitic diseases. Compared with a normal control group, these children do not have a subpopulation of

cells lacking surface immunoglobulin that mark with CD1a, CD2, CD3, CD4, and CD8. Which of the following karyotypic

abnormalities is most likely to be seen in the children in this study?

□ (A) +21

□ (B) 22q11.2

□ (C) t(9;22)

□ (D) t(15;17)

□ (E) X(fra)

□ (F) XXY

A

(B) These cells mark as cortical lymphocytes in the thymus of a child. An absence of such cells can be seen in the

DiGeorge anomaly with 22q11.2. Such patients also can have parathyroid hypoplasia and congenital heart disease.

Patients with Down syndrome (trisomy 21) can have thymic abnormalities and the T-cell dysregulation that predisposes to

acute leukemia, but the thymus is typically present. The t(9;22) gives rise to the Philadelphia chromosome, which is

characteristic of chronic myelogenous leukemia. The t(15;17) is seen in patients with acute promyelocytic leukemia.

Individuals with fragile X syndrome usually have some form of mental retardation. Males with Klinefelter syndrome (XXY)

do not have immunological abnormalities.

49
Q

A 38-year-old woman has experienced increasing dyspnea for the past 2 months. On physical examination, she is

afebrile and normotensive. Inspiratory wheezes are noted on auscultation of the chest. A chest CT scan shows an 8 ×

10 cm posterior mediastinal mass that impinges on the trachea and esophagus. A mediastinoscopy is performed, and the

mass is biopsied. Histologically, there are scattered large multinucleated cells, with prominent nucleoli that mark with

CD15, and lymphocytes and macrophages separated by dense collagenous bands. Which of the following is most likely to

be seen microscopically in this biopsy specimen?

□ (A) Atypical lymphocytes

□ (B) Histiocytes with Birbeck granules

□ (C) Hairy cells

□ (D) Lacunar cells

□ (E) Lymphoblasts

□ (F) Myeloblasts

A

(D) The lacunar cells and the CD15+ Reed-Sternberg cells indicate Hodgkin lymphoma, and the fibrous bands suggest

the nodular sclerosis type. Lacunar cells have multilobed nuclei containing many small nucleoli. These cells have

artifactual retraction of the cytoplasm around the nucleus, giving the cells their distinctive appearance. The nodular

sclerosis type of Hodgkin lymphoma is more common in women. Atypical lymphocytes are characteristic in the peripheral

blood of individuals with infectious mononucleosis. Histiocytes with Birbeck granules are characteristic of the Langerhans

cell histiocytoses. Hairy cell leukemia often is accompanied by splenomegaly, but not a mediastinal mass, and the

leukemic cells are B cells. Lymphoblasts that mark as T cells are seen in anterior mediastinal (thymic) masses in children

with acute lymphoblastic leukemia/lymphoma. Myeloblasts are characteristic of acute myelogenous leukemia, which is

occasionally accompanied by soft-tissue masses

50
Q

A 30-year-old, previously healthy man has had an enlarging nodular area on his arm for the past 8 months. On physical

examination, there is an ulcerated, reddish violet, 3 × 7 cm lesion on his right forearm and nontender right axillary and left

inguinal lymphadenopathy. A chest radiograph shows a 4-cm nodular left pleural mass. An abdominal CT scan shows a 5-

cm right retroperitoneal mass. Biopsy of an inguinal node is performed, and microscopic examination shows large

anaplastic cells, some of which contain horseshoe-shaped nuclei and voluminous cytoplasm. The tumor cells cluster

around venules and infiltrate sinuses. The patient goes into remission after chemotherapy. Which of the following

immunohistochemical markers is most likely to be positive in the tumor cells?

□ (A) Anaplastic lymphoma kinase protein

□ (B) CD10

□ (C) c-KIT proto-oncogene

Robbins & Cotran Review of Pathology Pg. 257

□ (D) HTLV-1

□ (E) p24 antigen

A

(A) This patient has a form of T-cell neoplasm known as anaplastic large-cell lymphoma, which most often appears in

children and young adults. It is often extranodal and has a characteristic gene rearrangement on chromosome 2p23 that

results in production of anaplastic lymphoma kinase (ALK) with tyrosine kinase activity. CD10 is a B-cell marker. The Tcell

proliferations involving skin, known as mycosis fungoides/Sézary syndrome, are CD4+. The c-KIT proto-oncogene has

been associated with some NK cell lymphomas. The p24 antigen is part of HIV, which is most often associated with B-cell

neoplasms.

51
Q

A 28-year-old man is brought to the emergency department with shock that developed over the past 12 hours. On

physical examination, his temperature is 38.6°C, pulse is 101/min, respirations are 19/min, and blood pressure is

80/40 mm Hg. Needle tracks are noted in the left antecubital fossa. Crackles are heard over the lower lung fields. CBC

shows hemoglobin, 14.1 g/dL; hematocrit, 42.6%; MCV, 93 μm3; platelet count, 127,500/mm3; and WBC count,

12,150/mm3 with 71% segmented neutrophils, 8% bands, 14% lymphocytes, and 7% monocytes. The neutrophils show

cytoplasmic toxic granulations and Döhle bodies. Which of the following is the most likely diagnosis?

□ (A) Pulmonary Mycobacterium tuberculosis

□ (B) Acute myelogenous leukemia

□ (C) Chronic myelogenous leukemia

□ (D) Pseudomonas aeruginosa septicemia

□ (E) Infectious mononucleosis

□ (F) Pneumocystis carinii pneumonia

A

(D) Toxic granulations, which are coarse and dark primary granules, and Döhle bodies, which are patches of dilated

endoplasmic reticulum, represent reactive changes of neutrophils. These changes are most indicative of overwhelming

inflammatory conditions, such as bacterial sepsis. The route of infection in this case is probably intravenous drug use.

Leukemia, granulomatous infections, or viral infections do not cause toxic changes in neutrophils. Infectious

mononucleosis is accompanied by an increase in “atypical” lymphocytes

52
Q

A 60-year-old man has had increasing abdominal discomfort and fatigue for the past 9 months. He has noted easy

bruising of his skin with minor trauma for the past month. On examination, he is afebrile, but his spleen is palpable and

tender. Laboratory studies show hemoglobin, 7.7 g/dL; hematocrit, 23%; platelet count, 30,000/mm3; and WBC count,

2300/mm3 with 45% polymorphonuclear leukocytes, 50% lymphocytes, and 5% monocytes. His serum AST is 71 U/L, and

ALT is 82 U/L. What is the most likely diagnosis?

□ (A) Acute myelogenous leukemia

□ (B) Cirrhosis

□ (C) Infectious mononucleosis

□ (D) Niemann-Pick disease

□ (E) Metastatic adenocarcinoma

□ (F) Systemic lupus erythematosus

A

(B) The cytopenias along with the splenomegaly suggest hypersplenism as a cause for this patient’s anemia,

leukopenia, and thrombocytopenia. The circulating blood cells are sequestered within the large spleen. One of the most

common causes is congestive splenomegaly from portal hypertension resulting from cirrhosis. His elevated AST and ALT

suggest liver disease. The WBC count should be quite high with acute leukemias. Although infectious mononucleosis can

lead to splenomegaly and can be marked by lymphocytosis, the course is typically not 9 months, and it is more common in

younger individuals. Niemann-Pick disease is a storage disease from an inborn error of metabolism involving

sphingomyelinase, and typically appears in childhood and leads to profound neurologic problems. The spleen is an uncommon location for metastases. Systemic lupus erythematosus can lead to cytopenias from reduced bone marrow function, but the spleen is not usually enlarged

53
Q

The figure skater Sonja Henie, who won gold medals at the 1928, 1932, and 1936 Winter Olympic Games, became

progressively fatigued in her late 50s. On physical examination, she had palpable nontender axillary and inguinal lymph

nodes, and the spleen tip was palpable. Laboratory studies showed hemoglobin, 10.1 g/dL; hematocrit, 30.5%; MCV,

90 μm3; platelet count, 89,000/mm3; and WBC count, 31,300/mm3. From the peripheral blood picture shown in the figure,

which of the following is the most likely diagnosis?

□ (A) Infectious mononucleosis

□ (B) Chronic lymphocytic leukemia

□ (C) Iron deficiency anemia

□ (D) Leukemoid reaction

A

(B) Sonja Henie died from complications of chronic lymphocytic leukemia (CLL), in which there are increased numbers

of circulating small, round, mature lymphocytes with scant cytoplasm seen in the peripheral blood smear. The cells

express the CD5 marker and the pan B cell markers CD19 and CD20. Most patients have a disease course of 4 to 6 years

before death, and symptoms appear as the leukemic cells begin to fill the marrow. In some patients, the same small

lymphocytes appear in tissues; the condition is then known as small lymphocytic lymphoma. The lymphocytes seen in

infectious mononucleosis are “atypical lymphocytes,” which have abundant, pale blue cytoplasm that seems to be

indented by the surrounding RBCs. The RBCs in iron deficiency anemia are hypochromic and microcytic, but the WBCs

are not affected. Leukemoid reactions are typically of the myeloid type, and the peripheral blood contains immature

myeloid cells. The WBC count can be very high, but the platelet count is normal. Acute lymphoblastic leukemia is a

disease of children and young adults, characterized by proliferation of lymphoblasts. These cells are much larger than the

cells in CLL and have nucleoli.

54
Q

A 48-year old man visits his physician for a routine health maintenance examination. He has no complaints other than

worrying about getting older and having cancer. Physical examination shows that he is afebrile and normotensive. There is

no hepatosplenomegaly or lymphadenopathy. Laboratory studies show a total serum protein level of 7.4 g/dL and albumin

level of 3.9 g/dL. Serum calcium and phosphorus levels are normal. Urinalysis shows no Bence Jones proteinuria.

Hemoglobin is 13.6 g/dL, platelet count is 301,500/mm3, and WBC count is 6630/mm3. A serum protein electrophoresis

shows a small (2.8-g) spike of γ-globulin, which is determined by immunoelectrophoresis to be IgG kappa. A bone marrow

biopsy specimen shows normal cellularity with maturation of all cell lines. Plasma cells constitute about 4% of the marrow.

A bone scan is normal, and there are no areas of increased uptake. What is the most likely diagnosis?

□ (A) Solitary plasmacytoma

□ (B) Waldenström macroglobulinemia

□ (C) Monoclonal gammopathy of undetermined significance

□ (D) Heavy-chain disease

□ (E) Multiple myeloma

□ (F) Reactive systemic amyloidosis

A

(C) Monoclonal gammopathy of uncertain significance (MGUS) is characterized by the presence of an M protein

“spike” in the absence of any associated disease of B cells. The diagnosis of MGUS is made when the monoclonal spike

is small (<3 g), and the patient has no Bence Jones proteinuria. MGUS can progress to multiple myeloma in about 20% of

patients over 10 to 15 years. A plasmacytoma would appear on a bone scan. Waldenström macroglobulinemia would be

accompanied by an IgM spike, hepatosplenomegaly, and lymphadenopathy. Heavy-chain disease is a rare condition that

can be seen in chronic lymphocytic leukemia. In multiple myeloma, the spike is greater than 3 g, and usually the patient

has bone lesions. In reactive systemic amyloidosis, serum amyloid-associated (SAA) protein derived from chronic

inflammatory conditions is deposited as AA amyloid in visceral organs, but there is no monoclonal gammopathy.

55
Q

A 32-year-old woman visits her physician because she has experienced fatigue, fever, night sweats, and painless

lumps in the right side of her neck for the past 3 months. On physical examination, her temperature is 37.5°C. She has

right cervical nontender lymphadenopathy. One of the lymph nodes is biopsied, and the histologic finding is shown at high

power in the figure. Molecular analysis of large cells exemplified by the cell of the center is most likely to reveal which of

the following genetic abnormalities?

□ (A) Clonal rearrangement of T-cell receptor genes

□ (B) Clonal rearrangement of immunoglobulin genes

□ (C) Polyclonal rearrangement of T-cell receptor genes

□ (D) Polyclonal rearrangement of immunoglobulin genes

□ (E) Integration of the cytomegalovirus genome

□ (F) Integration of the human herpesvirus-8 genome

□ (G) Integration of the HTLV-1 genome

A

(B) The cell at the center of the figure is a Reed-Sternberg (RS) cell, characteristic for Hodgkin lymphoma. Studies

have conclusively established that in most cases of Hodgkin lymphoma, RS cells are derived from B cells. Within a given

tumor, all RS cells have clonal (identical) immunoglobulin gene rearrangements. T-cell receptor gene rearrangements

occur in normal or neoplastic T cells (e.g., acute lymphoblastic lymphoma). In most cases of Hodgkin lymphoma, RS cells

contain the Epstein-Barr virus (EBV) genome, but not the CMV or HHV-8 genome. This latter genome is found in cells of

Kaposi sarcoma. HTLV-1 infects CD4+ T cells and gives rise to adult T-cell leukemia/lymphoma.

56
Q

A 22-year-old university student reports easy fatigability of 2 months’ duration. On physical examination, she has no

hepatosplenomegaly or lymphadenopathy. Mucosal gingival hemorrhages are noted. CBC shows hemoglobin, 9.5 g/dL;

hematocrit, 28.2%; MCV, 94 μm3; platelet count, 20,000/mm3; and WBC count, 107,000/mm3. A bone marrow biopsy

specimen shows that the marrow is 100% cellular with few residual normal hematopoietic cells. Most of the cells in the

marrow are large, with nuclei having delicate chromatin and several nucleoli. The cytoplasm of these cells has azurophilic,

peroxidase-positive granules. Which of the following is the most likely diagnosis?

□ (A) Acute lymphoblastic leukemia

□ (B) Acute myelogenous leukemia

Robbins & Cotran Review of Pathology Pg. 259

□ (C) Chronic lymphocytic leukemia

□ (D) Chronic myelogenous leukemia

□ (E) Hodgkin lymphoma

□ (F) Sézary syndrome

A

(B) The very high WBC count and the presence of peroxidase-positive blasts (myeloblasts) filling the marrow are

characteristic of acute myelogenous leukemia. This type of leukemia is most often seen in individuals 15 to 39 years old.

Acute lymphoblastic leukemia occurs in children and young adults. Azurophilic, peroxidase-positive granules distinguish

myeloblasts from lymphoblasts. Chronic lymphocytic leukemia is characterized by the presence of small, mature

lymphocytes in the peripheral blood and bone marrow of older adults. Chronic myelogenous leukemia also is seen in

adults, but this is a myeloproliferative process with a range of myeloid differentiation, and most of the myeloid cells are

mature, with few blasts. Hodgkin lymphoma does not have a leukemic phase. Some cutaneous T-cell lymphomas may

have circulating cells, known as Sézary cells.

57
Q

A 9-year-old, otherwise healthy girl has complained of pain in the right armpit for the past week. Examination by the

physician shows tender lymphadenopathy of the right axillary region. There are four linear and nearly healed abrasions

over a 3 × 2 cm area of the distal ventral aspect of the right forearm and a single, 0.5-cm, slightly raised erythematous

nodule over one of the abrasions. No other abnormalities are noted. If a lymph node biopsy were performed, the

microscopic appearance of the specimen would show a pattern of stellate, necrotizing granulomas. The lymphadenopathy

regresses over the next 2 months. Infection with which of the following is most likely to have produced these findings?

□ (A) Bartonella henselae

□ (B) Cytomegalovirus

□ (C) Epstein-Barr virus

□ (D) Staphylococcus aureus

□ (E) Yersinia pestis

A

(A) This child has cat-scratch disease, a form of self-limited infectious lymphadenitis that most often is seen in children,

typically “upstream” of lymphatic drainage from the site of injury, so that the axillary and cervical lymph node regions are

most often involved. Cytomegalovirus infection is typically seen in immunocompromised individuals and is not a common

cause of lymphadenopathy. Epstein-Barr virus (EBV) infection at this age is most often associated with infectious

mononucleosis and pharyngitis, and the lymphadenopathy is nonspecific. Staphylococcus aureus can produce

suppurative inflammation with sepsis. Yersinia pestis, the agent that causes bubonic plague, produces lymphadenopathy

that can ulcerate and a hemorrhagic necrotizing lymphadenitis; it has a high mortality rate.

58
Q

A 63-year-old man has noticed a lump in her neck for 2 months. Examination reveals a group of three discrete nontender

right posterior cervical lymph nodes, and a mass of enlarged right axillary lymph nodes. Chest and abdominal CT

scans show mediastinal lymphadenopathy and hepatosplenomegaly. A cervical lymph node biopsy reveals abundant large

CD15+ and CD30+ binucleate cells with prominent acidophilic nucleoli, scattered within a sparse lymphocytic infiltrate.

What is molecular analysis of this lesion most likely to reveal?

□ (A) Clonal EBV integration in the large cells

□ (B) BCL6 gene rearrangements in the large cells

□ (C) Deletions of 5q in all the cells

□ (D) Helicobacter pylori infection in all the cells

□ (E) JAK2 gene mutations in the lymphocytes

A

(A) This is the lymphocyte depletion variant of Hodgkin lymphoma with abundant Reed-Sternberg cells and paucity of lymphocytes. Most cases present with advanced disease (stage IV in this example). Epstein-Barr virus (EBV) is present in

over 90% of cases. BCL6 gene rearrangements are typical of diffuse large B-cell lymphomas. Deletions of 5q are typical of

myelodysplastic syndrome. Infection with Helicobacter pylori can give rise to marginal zone lymphoma. JAK2 mutations

are found in polycythemia vera and other myeloproliferative diseases.